Построить SO(3)SO(3)SO(3) вращение внутри двух SU(2)SU(2)SU(2) фундаментальных вращений

Мы знаем, что два С U ( 2 ) основы имеют разложение умножения, так что

(1) 2 2 "=" 1 3.
В частности, 3 имеет векторное представление С О ( 3 ) . В то время как 1 является тривиальным представлением С U ( 2 ) .

надеюсь увидеть точную С О ( 3 ) ротация из двух С U ( 2 ) фундаментальные вращения.

Итак, давайте сначала напишем два С U ( 2 ) фундаментальные объекты с точки зрения С О ( 3 ) объект. В частности, мы можем рассмотреть следующие три:

| 1 , 1 "=" ( 1 0 ) ( 1 0 ) "=" | ↑↑ ,
| 1 , 0 "=" 1 2 ( ( 1 0 ) ( 0 1 ) + ( 0 1 ) ( 1 0 ) ) "=" 1 2 ( | ↑↓ + | ↓↑ ) ,
| 1 , 1 "=" ( 0 1 ) ( 0 1 ) "=" | ↓↓ .

где | и находятся в С U ( 2 ) основы. И мы выстрелили | ↑↑ | | и так далее.

Вопрос: Как мы переключаемся между | 1 , 1 , | 1 , 0 , | 1 , 1 , через два С U ( 2 ) вращения, действующие на два С U ( 2 ) основы? А именно, то есть построить С О ( 3 ) вращение внутри двух С U ( 2 ) фундаментальные вращения? С U ( 2 ) имеет три генератора, параметризованных м Икс , м у , м г ; как мы запишем общий С О ( 3 ) вращения из двух С U ( 2 ) вращения?

Рассмотрим пример, С U ( 2 ) вращение U действующий на С U ( 2 ) фундаментальный ( 1 0 ) дать начало

U ( 1 0 ) "=" ( потому что ( θ 2 ) + я м г грех ( θ 2 ) ( я м Икс м у ) грех ( θ 2 ) ( я м Икс + м у ) грех ( θ 2 ) потому что ( θ 2 ) я м г грех ( θ 2 ) ) ( 1 0 ) "=" ( потому что ( θ 2 ) + я м г грех ( θ 2 ) ( я м Икс + м у ) грех ( θ 2 ) ) потому что ( θ 2 ) + я м г грех ( θ 2 ) ( 1 0 ) + ( я м Икс + м у ) грех ( θ 2 ) ( 0 1 )

Другими словами, С U ( 2 ) вращение U | м | 2 "=" 1 ) вращается С U ( 2 ) основы. Два С U ( 2 ) вращения действуют как

U U | 1 , 1 "=" U ( 1 0 ) U ( 1 0 ) "=" ( потому что ( θ 2 ) + я м г грех ( θ 2 ) ( я м Икс + м у ) грех ( θ 2 ) ) ( потому что ( θ 2 ) + я м г грех ( θ 2 ) ( я м Икс + м у ) грех ( θ 2 ) )

Подсказка: наивно, мы любим конструировать

л ± "=" л Икс ± я л у ,
такой, что л ± является оператором из двух С U ( 2 ) вращения, действующие на два С U ( 2 ) основы, так что он поднимается/опускается между | 1 , 1 , | 1 , 0 , | 1 , 1 .

вопрос 2: Возможно ли, что два С U ( 2 ) невозможно выполнить такие С О ( 3 ) вращения, но нам нужно два U ( 2 ) вращения?

Ответы (2)

Следующее решение исходит из теории геометрического квантования. Я не буду объяснять полную теорию, стоящую за этим, но я приведу здесь решение, а затем кратко расскажу, как проверить, является ли это требуемым решением.

Генерал С U ( 2 ) элемент группы в фундаментальном представлении можно записать как:

г "=" [ α β β ¯ α ¯ ]
с
| α | 2 + β | 2 "=" 1
Трехмерное гильбертово пространство трехмерного представления можно параметризовать следующим образом:
ψ ( г ) "=" а + б г + с г 2 ( 1 )

где Икс является неопределенным

Действие С U ( 2 ) в этом векторном пространстве определяется как:

( г ψ ) ( г ) "=" ( β ¯ г + α ¯ ) 2 ψ ( α г + β β ¯ г + α ¯ ) ( 2 )

  1. Чтобы убедиться, что это представление, можно проверить, что композиция действия двух групповых элементов совпадает с действием их произведения.
  2. Чтобы увидеть, что это верный С О ( 3 ) представительство, но не верный С U ( 2 ) , легко видеть, что для нетривиального элемента центра:
    г с "=" [ 1 0 0 1 ]
    У нас есть для каждого ψ
    ( г с ψ ) ( г ) "=" ψ ( г )
  3. Хотя, "сферические" компоненты а , б , с являются сложными. Чтобы убедиться, что представление реально, можно увидеть, что «декартовы» компоненты ( а + с ) , б , я 1 ( а с ) преобразовывать с помощью только действительных комбинаций α и β .
@ Давид Бар Моше, спасибо за ответ, +1. Ваш ответ находится на более продвинутом уровне. :)
Тогда я думаю, может быть, вы также можете решить это или, по крайней мере, поделиться своим мнением: math.stackexchange.com/q/2745276 - спасибо.
@ Дэвид, это легко увидеть из
U U | 1 , 1 "=" U ( 1 0 ) U ( 1 0 ) "=" ( потому что ( θ 2 ) + я м г грех ( θ 2 ) ( я м Икс + м у ) грех ( θ 2 ) ) ( потому что ( θ 2 ) + я м г грех ( θ 2 ) ( я м Икс + м у ) грех ( θ 2 ) )
когда м у "=" ± 1 , м Икс "=" м г "=" 1 и θ "=" π , у нас есть
U U | 1 , 1 "=" ( 0 1 ) ( 0 1 ) "=" | 1 , 1
У вас есть точный ответ, как найти U , такой, что
U U | 1 , 1 "=" 1 2 ( 1 0 ) ( 0 1 ) + 1 2 ( 1 0 ) ( 0 1 ) "=" | 1 , 0 ?
@annie сердце такого нет U . Пожалуйста, обратите внимание, что если вы действуете по какому-либо U U на вектор старшего веса: ( 1 0 ) ( 1 0 ) , то результатом всегда будет сепарабельный вектор вида ( а б ) ( а б ) . Но вектор, которого вы хотите достичь, запутан, и нет никакого способа изменить состояние запутанности с помощью локального преобразования.
Действительно, структура запутанности другая, тогда как насчет построения поворотов из двух поворотов U(2) вместо двух SU(2)?
Нет, U ( 2 ) также не изменит структуру запутанности, An С U ( 3 ) потребуется элемент, действующий на трехмерное векторное пространство.

Возможно, будет полезно следующее:

  1. Экв. ОП. (1) следует понимать как отношение между комплексными представлениями С U ( 2 ) , т.е. комплексные векторные пространства. Напоминая, что основополагающим С U ( 2 ) представление 2 2 ¯ изоморфно комплексно-сопряженному представлению, вместо этого рассмотрим изоморфизм

    (А) 2 2 ¯     1 3 .

  2. Левая часть уравнения. (A) может быть реализовано как вещественное векторное пространство ты ( 2 ) из 2 × 2 Эрмитовы матрицы. Группа С U ( 2 ) действует на ты ( 2 ) через сопряжение. Учитывая спинор | ψ е 2 , затем

    (Б) 1 с ты ( 2 )     ты ( 2 )     | ψ ψ |   "="   1 2 мю "=" 0 3 Икс мю о мю , ( Икс 0 , Икс 1 , Икс 2 , Икс 3 )   е   р 4 .
    Тройка 3 соответствует бесследовой части, то есть: с ты ( 2 ) . Следовательно, спинор | ψ представляет 3-вектор р "=" ( Икс 1 , Икс 2 , Икс 3 ) . См. также этот пост на Phys.SE.

Спасибо! +1, тот же вопрос, что и к Дэвиду, есть ли у вас точное вращение от того, как найти U, такое, что UU|1,1⟩=(1/√2) (10)(01)+(1/√2) √(10)(01)=|1,0⟩ некоторым U?